User Tools

Site Tools


eugenics

Title Pass the Speed, ggz nl

Same text here green is speed

RIVIERDUINEN LEIDSCHENDAM

ALL RUYT.4 GGZ ROOMS:

HAVE…

o green light,

SEE ILLUSTR !!

O WEIRD ALIGN

===

O PRB POLAROID

O SO EYE GETS LESS LIGHT

O EYE HOLE OPENS

O THEN RULE: LESS DEPTH

O THEN OBJ HAVE HIGHER

VIRTUAL SPEED WHEN PASSING

Hence the title

O TIRING THOUGH VIRTUAL

===

Illustratie 1/2

Zoals je ziet…

Onze lampen zijn (Iig) ook groen

20240721_045509.jpg

===

Uitleg ××

O GREEN LIGHT

O POLARIZED

NOT HEAT, THEN /BEAM/

A TYPICAL SZ DELUS

MINE AT LEAST

BEAM OF VOLVO ACC RADAR

===

There's more

A hidden beam and tiring… already problematic

But from scifi we learn:

It's not research or maltreatment on patients

It's readying to make clients make a kill

This can be illustrated from the Movie Immortals

And there is a simple way…

So you sleep under freaky hidden but green light beams

Well there s no one who wants green, we just happen to relax

We want… blood

Yes RED

so there's a Dayak experiencing virtual swimming speed in a 3d washing machine.

This puts it in a state to smell and kill

Illustr 2/2 the victims are slow and lack smell

screenshot_20240721_054812_firefox.jpg

===

ALL

TY

FEEL FREE TO REFER TO THIS

===

××HIGHLIGHTED FROM:

https://physics.stackexchange.com/questions/302795/why-does-stacking-polarizers-of-the-same-angle-still-block-more-and-more-light

Fair use Used for inspect Of polar. In mental Health hospital

Physics

Why does stacking polarizers of the same angle still block more and more light? Asked 7 years, 6 months ago Modified 7 years, 6 months ago Viewed 4k times 18

I have some sheets of polarization film. They came in a big box, all stacked at the same angle. I noticed that the entire stack of them lets almost no light through, even though they're all at the same angle.

I pulled out two, and those two also block more light than just one.

Why?

Is this because I have low-grade polarizers? Or because lining them up at EXACTLY the same angle is impossible? Or because the light that gets through the first one is not really polarized exactly to its angle — it's just that less of it is polarized away from its angle than before?

If it's because these are low-grade polarizers, can anyone recommend a linear polarizer that I can stack several of in a row at the same angle and still have a 100% probability of the light getting through?

I feel like I'm probably just misunderstanding polarization theory so please correct me.

  opticsvisible-lightpolarization

Share Cite Improve this question Follow edited Jan 4, 2017 at 19:18 user98038 asked Jan 4, 2017 at 4:24 CommaToast's user avatar CommaToast 46444 silver badges1414 bronze badges Add a comment 4 Answers Sorted by: 20

  Why? Is this because I have low-grade polarizers?

As any substance, the transmitance (amount of light that passes from one side to the other) of polarizers is not 100%. It depends on the materials and even if you can perfectly align the polarizers, it will not reach 100%.

An important aspect is that transmitance is dependant on the wavelength. So, in your case, your polarizers might not have a very high transmitance in visible light.

As always, if this is a good thing or not, it depends on what you will use it for. The table below indicates a type of polarizer for each priority characteristic. [2]

Prioritized aspect of polarizer and indicated type

  If it's because these are low-grade polarizers, can anyone recommend a linear polarizer that I can stack several of in a row at the same angle and still have a 100% probability of the light getting through?

100% is not feasible, as it has been said. But if you need higher transmission than you have you should probably choose dichroic glass, as indicated. Downsides are the price (this kinds of polarizer reaches “one thousand dollars for a polarizer with a one inch diameter” according to the document cited), they can not be very large due to manufacturing restrictions and they convert the unwanted light into heat.

[2] https://www.google.co.jp/url?sa=t&rct=j&q=&esrc=s&source=web&cd=7&cad=rja&uact=8&ved=0ahUKEwjU6s3g26fRAhWDiZAKHXnNAHkQFghKMAY&url=http%3A%2F%2Fwww.meadowlark.com%2Fstore%2FPDFs%2FPolarizer_Article_010109_StandardRes.pdf&usg=AFQjCNGz5020o5c9PWbn7LkijhHIbKaAeQ Share Cite Improve this answer Follow answered Jan 4, 2017 at 5:31 Odano Naotake's user avatar Odano Naotake 36611 silver badge55 bronze badges

  "they convert the unwanted light into heat." - what do other polarizers convert it to? – 
  John Dvorak
  Commented Jan 4, 2017 at 15:05
  @JanDvorak A polarizing beam splitter is a way of polarizing light without producing heat. It simply puts light of the unwanted polarization into a separate beam. – 
  Yly
  Commented Jan 4, 2017 at 20:12
  @Yly: But don't all optical media (not just polarizers) convert some light into heat? – 
  user21820
  Commented Jan 5, 2017 at 15:46

Add a comment 10

A high-quality supplier of polarizers and other optical equipment would be able to offer you data on the transmission characteristics of even their cheapest polarizers:

Transmission curves for Thorlabs 'economy' polarizers

I interpret this plot to mean that if you bought two of these devices, aligned their axes parallel to each other, and shone unpolarized λ=550nm green light on them, you'd only get 40% of your intensity out of the first polarizer, and of that fraction something like 10−4 still has the “wrong” polarization. From the second polarizer you'd only get 80%⋅40%=32%

of the original intensity, and another transmission factor 0.8 from any subsequent, also-aligned polarizers. It might be possible to improve the polarization by having multiple parallel filters, at the cost of this lost overall intensity, but you might also run into sneaky laboratory issues. Share Cite Improve this answer Follow answered Jan 4, 2017 at 8:25 rob's user avatar rob♦ 91.5k2020 gold badges172172 silver badges340340 bronze badges

  1
  As you mention, it the fact that the first polariser 'converts' unpolarised light to polarised light that does the majority of the perceived attenuation. For an interesting test (for the OP who has the sheets..), take the first two sheets and arrange them as a crossed polariser at maximum extinction, then insert a third polariser between them at a 45 degree angle. What happens and Why... ? – 
  Philip Oakley
  Commented Jan 4, 2017 at 16:08
  I guess "unpolarized" light has equally distributed random polarization around a circle O. Polarization lets thru the light according to probability (cos 0-θ)^2 where θ was the light's polarization when it hit. Then if the light next goes through a 90-degree polarizer, it will now have a probability (cos 90-θ)^2 of being polarized at an angle θ. Since the waves y = cos 0 and y = cos 90 are opposite phase, they cancel and no light gets through. But with (cos 45-θ)^2 in between, the probability waves don't cancel out. Is that basically correct or no? – 
  CommaToast
  Commented Jan 4, 2017 at 23:41
  @CommaToast: Unpolarized light has random polarization, but the polarization of light behaves as a vector. Passing light through a filter will pass through the vector component that's parallel to the filter, while removing the component that's perpendicular. – 
  supercat
  Commented Jan 4, 2017 at 23:56
  @supercat I think that's basically what I'm saying, because the vector you speak of represents the probability amplitude (i.e. cosine) of the angle of polarization. In other words the polarization vector is not the actual direction of polarization of the actual photons; it's just a representation of probabilities having to do with their actual angles of polarization having enough of a component in the direction of the polarizer's angle that they can pass through it (after which their actual angles of polarization could have changed to some degree, for all we know). Right? – 
  CommaToast
  Commented Jan 5, 2017 at 0:25 
  @supercat: And it attenuates the component parallel to the filter as well while it's at it. – 
  user21820
  Commented Jan 5, 2017 at 15:49

Show 2 more comments 5

In addition to the other answers: Even if you had extremely clear glass (like for optical fibers), stacking them would cause them to get more quickly opaque than what would be expected by their transmission coefficient. This is only for completeness because the effect on real polarized filters is dominated by their transmission coefficient as said by Odano.

The reason is that on every boundary surface (seam between two glasses) light is reflected, the amount is approximately 4%. So after 17 glasses only 50% of light is transmitted even with perfectly clear glass. Share Cite Improve this answer Follow edited Jan 4, 2017 at 18:59 tomph's user avatar tomph 71266 silver badges1515 bronze badges answered Jan 4, 2017 at 18:32 Thorsten S.'s user avatar Thorsten S. 2,0201212 silver badges1717 bronze badges Add a comment 3

An ideal (theoretical) polarizer will only let in light along a certain axis.

This is impossible in real life. Any polarizer that you can purchase will let in light in a range of possible polarization angles. Additionally, manufacturing processes cannot guarantee perfect alignment with the apparatus holding the polarizer.

Therefore, when you are aligning them all up, you are not actually accomplishing that. They are still out of alignment, even if in small amounts. Share Cite Improve this answer Follow answered Jan 4, 2017 at 4:33 occamsrazor's user avatar occamsrazor 9944 bronze badges

  So once the light comes out of the polarizer, it's fair to say that its polarization angle is not exactly the same as the angle of the polarizer? But that we can be pretty sure it's definitely not the same as an angle perpendicular to the polarizer? It seems to me that some light in a decreasing range of density across other angles must be getting through, since otherwise inserting a 45° polarizer between a 0° and a 90° would reduce the light by less than 1/2 again as much as the first one. – 
  CommaToast
  Commented Jan 4, 2017 at 6:16 
  It seems to me it must be like Trump combing his hair. The more he combs it the straighter it gets but the more falls out. It's never perfectly straight though, even if he thinks "it's the straightest." – 
  CommaToast
  Commented Jan 4, 2017 at 6:18
  @CommaToast You've got the right idea. It's not that the light doesn't align with the polarizer, it's that the polarizer doesn't have a strict, perfect axis through which it lets light through. In other words, the angle of the polarizer has an uncertainty in it. As the relative angles of the polarizers becomes large, this uncertainty becomes small, so it perfectly reasonable to say that there probably isn't much light going through two polarizers at 90°. – 
  occamsrazor
  Commented Jan 4, 2017 at 6:36
  1
  Careful, you can have reflective perfect polarizers with Brewster's angle. – 
  Thorsten S.
  Commented Jan 4, 2017 at 18:23

Add a comment Highly active question. Earn 10 reputation (not counting the association bonus) in order to answer this question. The reputation requirement helps protect this question from spam and non-answer activity. Not the answer you're looking for? Browse other questions tagged

  opticsvisible-lightpolarization

or ask your own question.

  Featured on Meta
  Announcing a change to the data-dump process
  Upcoming initiatives on Stack Overflow and across the Stack Exchange network...
  We spent a sprint addressing your requests — here’s how it went

Related 6 Why do everyday plastic items interfere with light polarizers? 0 Light intensity through polarizers and a diffuse reflection 9 Circular polarizers change the color of linearly polarized light 2 Does polarized light reflect in the same polarization? 0 Do polarizers also shift the phase of light? 2 Is “non-polarized” light made up of equal amounts of circularly polarized clockwise and counter-clockwise light? 2 What happens to circularly polarized light when it hits a linear polarizer? Hot Network Questions

  Why are some elves royalty?
  Is it possible to have a double miracle Sudoku grid?
  "Four or six times", where is five?
  Old client wants files from materials created for them 6 years ago
  Left crank arm misaligned on climb
  I can't find a nice literal translation for "Stella caelis exstirpavit"
  Swap the positions of the 4 chess knights
  Reorder for smallest largest prefix sum
  Optoisolated and level-shifted Esp32 Input
  Is this circuit safe to put in my ceiling? What improvements could I make?
  How to access specific entry from list of arguments, when index is provided as a letter?
  Accelerating semidecision of halting problem
  Is "farfel" an idiolectical quirk/part of a familect?
  Why don't we call value investing "timing the market"?
  Story about 2 people who can teleport, who are fighting, by teleporting behind the each other to kill their opponent
  Is it rude to ask Phd student to give daily report?
  When can widening conversions cause problems?
  Does Ephesians 4:11 have any implication regarding the Second Coming of Christ?
  What are the functions obtained by complex polynomials evaluated at complex numbers
  Were ancient Greece tridents different designs from other historical examples?
  Does the grammar underlying 为期 generalize to 为主, 为基础, and so on?
  Alternative to isinglass for tarts or other desserts
  Examples of distributions with easily solvable quantile functions but hard to solve CDFs
  Does the variety of Boolean Algebras contain no proper nontrivial subvarieties/subquasivarieties?

Question feed

Physics

  Tour
  Help
  Chat
  Contact
  Feedback

Company

  Stack Overflow
  Teams
  Advertising
  Collectives
  Talent
  About
  Press
  Legal
  Privacy Policy
  Terms of Service
  Cookie Settings
  Cookie Policy

Stack Exchange Network

  Technology
  Culture & recreation
  Life & arts
  Science
  Professional
  Business
  API
  Data
  Blog
  Facebook
  Twitter
  LinkedIn
  Instagram

Site design / logo © 2024 Stack Exchange Inc; user contributions licensed under CC BY-SA. rev 2024.7.18.12879

eugenics.txt · Last modified: 2024/07/21 03:57 by adminm

Donate Powered by PHP Valid HTML5 Valid CSS Driven by DokuWiki